The function y = f(x) is graphed below. What is the average rate of change of the
function f(x) on the interval -8 ≤ x ≤ 8?

The Function Y = F(x) Is Graphed Below. What Is The Average Rate Of Change Of Thefunction F(x) On The

Answers

Answer 1

The average rate of change of the function is -35/16.

How to find the average rate of change of the function?

We have to determine the average rate of change of the function f(x) on the interval -8 ≤ x ≤ 8.

so

at x₁ = -8, f(x₁) = f(-8) = 30

at x₂ = 8, f(x₂) = f(8) = -5

Average rate = [f(x₂) - f(x₁)] / [ x₂ - x₁]  

                     = [f(8) - f(-8) ] / [8-(-8)]

                     = [-5 - (30)] / [16]

                     = [-35] / 16

Therefore, the average rate of change of the function is -35/16.

Learn more about the average rate;

https://brainly.com/question/21250694

#SPJ1


Related Questions

Is (−1, −1) a solution to the following system of inequalities? Show work to verify algebraically. Then, provide an explanation below.

Answers

Yes this is a solution.

A way you can figure this out is by plugging in (-1, -1) to their corresponding values in both equations. The first -1 would replace all x values. The second -1 would replace all Y values.

After simplifying the first equation gives
-1 >= -4 + 3
-1 >= -1 TRUE

The second equations gives

-1 - (-2) >= 1
1 >= 1 TRUE

Select the correct answer. Simplify the polynomial expression. 3x(-2x+7) -5(x-1)(4x-3)

Answers

Answer:

The simplified form is -26x^2 + 56x -15

Step-by-step explanation:

We need to solve the expression:

3x(-2x+7)-5(x-1)(4x-3)

Multiplying the terms outside the bracket with the terms inside the bracket.

=-6x^2+21x-5(x(4x-3) -1(4x-3))

= -6x^2+21x-5(4x^2-3x-4x+3)

= -6x^2+21x-5(4x^2-7x+3)

Now multiply -5 with the terms inside the bracket

= -6x^2+21x -20x^2 +35x -15

Now, Combining the like terms:

= -6x^2 -20x^2 +21x+35x -15

Adding the like terms

= -26x^2 + 56x -15

So, the simplified form is -26x^2 + 56x -15

-26^2+56x-15 yea that should be the answer

In the equation above. K is a constant. If x=9, what value is k? A) 1 B) 7 C) 16 D) 79

Answers

Answer: 79

Step-by-step explanation:

[tex]\sqrt{k+2}-9=0\\\\\sqrt{k+2}=9\\\\k+2=81\\\\k=\boxed{79}[/tex]

Answer:

D) k = 79

Step-by-step explanation:

Given equation: [tex]\sf\sqrt{k+2}-x=0[/tex]

Steps:

1. Substitute 9 as the value of x in the equation:

[tex]\sf\sqrt{k+2}-9=0[/tex]

2. Add 9 to both sides:

[tex]\sf\sqrt{k+2}-9+9=0+9\\\\\Rightarrow \sqrt{k+2}=9[/tex]

3. Square both sides of the equation:

[tex]\sf\left(\sqrt{k+2}\right)^2=9^2\\\\\Rightarrow k+2=81[/tex]

4. Subtract 2 from both sides:

[tex]\sf k+2-2=81-2\\\\\Rightarrow k = 79[/tex]

5. Check your work:

[tex]\sf\sqrt{k+2}-x=0\ \textsf{[ substitute 79 for k, and 9 for x ]}\\\\\sqrt{79+2}-9=0\ \textsf{[ add ]}\\\\\sqrt{81}-9=0\ \textsf{[ take the square root ]}\\\\9-9=0\ \textsf{[ subtract ]}\\\\0=0\ \checkmark[/tex]

A mapping diagram showing a relation, using arrows, between input and output for the following ordered pairs: (negative 3, negative 9), (2, negative 6), (negative 5, 4), (1, 2), (6, 0).
What is the domain of the function shown in the mapping?

Answers

Answer:

{-3, 2, -5, 1, 6}

Step-by-step explanation:

The domain is the set of input values.

Rewrite the rational exponent as a radical exponent ^{^3\sqrt{2^7}}

Answers

Answer:

4 ^3sqrt(2)

Step-by-step explanation:

two 2's can come out of the square root and it leaves one left in it and 2x2 is 4 which is on the outside.

Answer: 4^3sqrt(2)
Explanation: have to do step by step

Can someone take a look at this image and answer please!

Answers

The answers to the following question is

1) KL = 10

2) TE= 15 cm

3) angle PUT= 13

4) angle SQP = 21

What is similarity in triangle?

Two triangles are similar if they have the same ratio of corresponding sides and equal pair of corresponding angles. If two or more figures have the same shape, but their sizes are different, then such objects are called similar figures.

1) Using similarity property in given triangle

SR/ KI= SJ / JK

AS, JK= 2 SJ

SR/ KI= SJ / 2 SJ

5/ KI =  1/2

KL= 10

2) As, the diagonal of parallelogram bisect equally each other then

VE = TE

AS, VE = 15

So, TE= 15 cm

3) As, PU is the bisector of angle SUT

angle PUT= 1/2 (angle SUT)

PUT = 1/2 (26)

angle PUT= 13

4) As PQ is the bisector SQR

angle PQR= angle SQP = 21

Learn more about similarity of triangles

https://brainly.com/question/25882965

#SPJ1

Need help to understand how to do this.

Answers

Answer: -1(x+2)^2+10

Step-by-step explanation:


1. The coefficient of -x^2 is just the number in front, which in this case would be a = -1 So now the function is -1(x^2+4x)+6 since negative one has been factored out of the first two terms.

2. Half of the coefficient of x would be 2 since x has a coefficient of 4, and half pf 4 is 2. Squared, it is 4. That will be added inside the bracket. Subtracted on the outside is the same equation but multiplied by a, which we found out was -1 in the beginning. So, the function becomes -1(x^2+4x+4)+6-(-4).

3. Factor the inside equation to find that it’ll become (x+2)^2, and simplify the outside to get 10. The function will now be;


f(x)=-1(x+2)^2+10


Hope that made sense and make sure to check just in case I did the math wrong :]

four students drew four different triangles in math class Anna triangle Yuna triangle Carly triangle saki triangle​

Answers

The question was incomplete. Below you will find the missing image.

Only the Saki's triangle can be solved using law of cosines.

Law of cosines
formula is used to find missing including angle or missing side.
It is used in SAS and SSS triangles.
If a, b & c are three sides and C be the vertices opposite to side c then,
c²=a²+b²-2ab×CosC
Here in the given image we see that,
In Anna's triangle, there is one side and one angle.
So, it is not possible to solve with law of cosines.
In Yuna's triangle, there are two angles and one side.
So, also it is not possible to solve with law of cosines.
In Carley's triangle, there are three angles.
Also in this case, law of cosines it not possible to use.
In Saki's triangle, there are three sides are given. So, it is a SSS triangle.
We can apply the law of cosines here in this triangle.

Learn more about law of cosines here :
https://brainly.com/question/4372174
#SPJ10

On Monday, a packaging company put together 450x packages of 45 cashews. On Tuesday, the company put together 175 less packages with 15x more cashews in each bag. Which of the following equations could be used to determine how many cashews were packaged on Tuesday?

Answers

Answer:

275x packages of 720 cashews

Step-by-step explanation:

450-175=275

(a+1)b

a=15 and b=45

(15 + 1) x 45 = 720

Describe the
translation.
y = (x - 5)² +5 →
y = (x −0)² +0
A.T<5,-5>
OB.T<-5,-5>
OC.T<-5,5>
OD. T<5,5>

Answers

The translation of the parabola from y = (x – 5)² + 5 to y = (x − 0)² + 0 will be (-5, -5). Then the correct option is B.

What is the parabola?

It's the locus of a moving point that keeps the same distance between a stationary point and a specified line. The focus is a non-movable point, while the directrix is a non-movable line.

The equation of a quadratic function, of vertex (h, k), is given by:

y = a(x – h)² + k

where a is the leading coefficient.

The translation of the parabola is given below.

y = (x – 5)² + 5 → y = (x − 0)² + 0

Then the translation will be (-5, -5).

Then the correct option is B.

More about the parabola link is given below.

https://brainly.com/question/8495504

#SPJ1

helppppp will give brainliest

Determine whether the relationship is an inverse variation or not. Explain.

a The product xy is not constant, so the relationship is an inverse variation.

b The product xy is constant, so the relationship is not an inverse variation.

c The product xy is constant, so the relationship is an inverse variation.

d The product xy is not constant, so the relationship is not an inverse variation

Answers

Answer:

The product xy is constant, so the relationship is an inverse variation.

Step-by-step explanation:

Inverse variation is the mathematical relationship between two variables which can be expressed by an equation in which the product of two variables is equal to a constant.

inverse variation is the relationships between variables that are represented in the form of y = k/x, where x and y are two variables and k is the constant value. It states if the value of one quantity increases, then the value of the other quantity decreases.

Here, the product xy is constant and gives the product as 960.

The relationship is inverse relationship.

Learn more about inverse  variation - https://brainly.com/question/2798700

#SPJ10

Q2 - Using fractions
Jarvis works in a garage for $7 an hour.
If he works on Saturday he is paid time and a quarter.
If he works on Sunday he is paid time and three quarters.
Last weekend Jarvis worked for four hours on Saturday and four hours on Sunday.
How much was Jarvis paid last weekend altogether?

Answers

Total Earned Saturday: $45

Total Earned Sunday: $63

Total: $108

Time and a quarter means that you divide his normal rate by 4 and add that to his normal rate.

9/4 = 2.25     2.25 + 9 = 11.25

Jarvis makes $11.25 an hour on Saturdays.

Since he worked 4 hours, you multiply,

11.25 x 4 = 45.

What is the normal rate?

Time and three quarters mean that you multiply his normal rate by 3/4 and add that to his normal rate

9x0.75 = 6.75     6.75 + 9 = 15.75

Jarvis makes $15.75 an hour on Sundays. Since he worked 4 hours, you multiply 15.75 x 4 = 63.

To get the total, you add his earnings on Saturday and earnings on Sunday

45 + 63 = 108.

To learn more about the using fractions visit:

https://brainly.com/question/78672

#SPJ1


Determine if the sequence 4, 10, 19, 31,... is arithmetic. If it is, determine the first term a and the common difference d

Answers

Answer:

It is not arithmetic, so therefore there is no common difference

Step-by-step explanation:

An arithmetic sequence is a sequence where each term increases by adding/subtracting some constant k. There is no constant term in the above pattern.

Answer:

  not an arithmetic sequence

Step-by-step explanation:

An arithmetic sequence is a sequence of numbers that have a common difference. That is, the difference between any term and the previous term is constant for the sequence. Other kinds of sequences have other relationships between the differences.

Differences

The "first" differences of this sequence are ...

10-4 = 619-10 = 931 -19 = 12

The first differences are not constant. However, we notice the "second" differences are constant. These are the differences of successive first differences.

9 -6 = 312 -9 = 3 . . . . . . constant 2nd differences

Sequence type

The first differences are not constant, so this sequence is not an arithmetic sequence.

For polynomial sequences, the level of constant difference tell you the degree of the polynomial describing the sequence. This sequence has constant 2nd-level differences, so can be described by a 2nd degree (quadratic) polynomial:

  f(n) = 1.5n² +1.5n +1 . . . . . a quadratic sequence

__

Additional comment

Sequences that are exponential have differences that have a common ratio. That ratio is the same at every level. It is the base of the exponential function.

5)Express in standard form: (i)4809000 (ii)0.00849​

Answers

Answer:

I hope it helps .in case you don't understand any part ,feel free to ask.

How do you solve this?

Answers

Step-by-step explanation:

to "solve" this I need an equation.

this whole expression must be equal to something.

without that I can only try to simplify the expression.

remember that

a/b / c/d = ad / bc

so, here we have

3a/(((a²/x) - 1)(a/x - 1)) = 3a/(a³/x² - a²/x - a/x + 1) =

= 3a/(a³/x² - a²/x - a/x - a/a) =

= 3a/((a²/x² - a/x - 1/x - 1/a)×a) = 3/(a²/x² - a/x - 1/x - 1/a)

can someone please help mee (20 points and i will give brainliest!!!)

Answers

Answer:

Step-by-step explanation:

Write an equation for the line parallel to the given line that contains C.
C (1,8); 5/7x + 7

Answers

-------------------------------------------------------------------------------------------------------------

Answer:  [tex]\textsf{y = 5/7x + 51/7}[/tex]

-------------------------------------------------------------------------------------------------------------

Given: [tex]\textsf{Goes through (1, 8) and is parallel to y = 5/7x + 7}[/tex]

Find:  [tex]\textsf{Write an equation that follows that criteria}[/tex]

Solution: We know that our equation is going to parallel to the line that was given therefore the slope would stay the same at 5/7.  We also have a point so we can plug in the values into the point-slope form, distribute, and solve for y.

Plug in the values

[tex]\textsf{y - y}_1\textsf{ = m(x - x}_1\textsf{)}[/tex][tex]\textsf{y - 8 = 5/7(x - 1)}[/tex]

Distribute

[tex]\textsf{y - 8 = (5/7 * x) + (5/7 * (-1))}[/tex][tex]\textsf{y - 8 = 5/7x - 5/7}[/tex]

Add 8 to both sides

[tex]\textsf{y - 8 + 8 = 5/7x - 5/7 + 8}[/tex][tex]\textsf{y = 5/7x - 5/7 + 8}[/tex][tex]\textsf{y = 5/7x + 51/7}[/tex]

Therefore, the final equation that follows the description that was provided in the problem statement is y = 5/7x + 51/7.

Which is the graph of the equation y-1=2/3(x-3)?

Answers

Answer:

The right graph

Step-by-step explanation:

Because the equation is in point slope form, we know it passes through (3,1) and has a slope of 2/3.

The set of life spans of an appliance is normally distributed with a mean mu = 48 months and a standard deviation sigma = 8 months. what is the z-score of an appliance that stopped working at 64 months?

Answers

The z-score of an appliance that stopped working at 64 months is 2.

What is mean?

The mean of observations is equal to the ratio of sum of all the observations and the number of observations.

Given, the set of life spans of an appliance is normally distributed with a mean mu = 48 months and a standard deviation sigma = 8 months

The z-score is then calculated as

z = x - μ /σ

For an appliance that stopped working at 64 months, we have

x = 64

On substituting the values, we get

z = 64-48/8

z = 2

Hence, the z-score of an appliance that stopped working at 64 months is 2

Learn more about mean.

https://brainly.com/question/11822836

#SPJ1

Answer:

D on edge

Step-by-step explanation:

2


Parallelogram EASY is drawn with diagonal ES. The measure of Angle AES is 40 degrees and the measure of Angle Y is 110
degrees.
Find the measure of Angle A, Angle YEA, Angle ESA, and Angle ESY.

Answers

Answer:

Step-by-step explanation:

Givens

Parallelogram EASY with diagonal ES

<AES = 40

<Y = 110

Solution

<AES = 40               GIVEN

<ESY = 40                     Z THEOREM OF A PRALLELOGRAM

<A = <Y                         PROPERTY OF A PARALLELOGRAM

<A = <110                      <A = 110 BECAUSE <Y = 110

<YES = 180 - <Y           -<ESY EVERY TRIANGLE HAS 180o

<YES = 180 - 110 - 40    SIMPLIFY

<YES = 30

<YEA = <YES + <AES    WE KNOW BOTH ANGLES ON THE RIGHT.

<YEA = 30 + 40             COMBINE

<YEA = 70

You could do <YEA by noting that consecutive angles of a Parallelogram equal 180

3 x 2/5 converted into a mixed number

Answers

Answer:

[tex]1 \frac{1}{5} [/tex]

Step-by-step explanation:

[tex]3 \times \frac{2}{5} \\ \frac{3}{1} \times \frac{2}{5} = \frac{6}{5} \\ \frac{6}{5} = 1 \frac{1}{5} [/tex]

Hi can someone assist me with this question and help me solve it?

Answers

Answer:

40°

Step-by-step explanation:

Given l \\ m,

m∠13 = m∠7 (alternate interior angles)

m∠13+m∠15 = 180° (Sum of angles in a straight line)

[tex]6x+4+(14x-4)=180\\6x+4+14x-4=180\\20x=180\\x=\frac{180}{20} \\=9\\\\[/tex]

Substitute x to find m∠13

m∠13= 6x+4 = 6(9)+4 = 36 + 4 = 40°

What is the multiplicative rate of change of the function? two-thirds three-fourths four-thirds three-halves

Answers

The multiplicative rate of change is 2/3

How to determine the multiplicative rate of change?

The complete question is in the image

The multiplicative rate of change is then calculated as:

r = y2/y1

This gives

r = 4/6

Simplify

r = 2/3

Hence, the multiplicative rate of change is 2/3

Read more about rate of change at:

https://brainly.com/question/4319809

#SPJ1

Answer:

b.

Step-by-step explanation:

Given the matrices A and B shown below, find -A + 1/3B

Answers

Step-by-step explanation:

1. first multiply -1 times all elements of matrix A

2. then multiply 1/3 by all elements of matrix B

3. then add each corresponding entries to get the result.

from step 1. matrix A will be

-4 -2. -1. -3

-2. 0. 1. -3

step 2. matrix B will be

3. -1. -2. -4

3. -10. 10. -1

add each corresponding elements to get

-1. -3. -3. -7

1. -10 11. -4

A couple plans to have 4 children. The gender of each child is equally likely. Design a simulation involving 55 trials that you can use to model the genders of the children. Write your answer as number

Answers

Answer:

4x55x2

Step-by-step explanation:

4 kids 55trys 50 50 odds

q(t)= Q_0 e^-kt where Q represents the quantity remaining after t years and k is the decay constant 0.00043. How long will it take for 500g of radium to decay to 5g?

Answers

It takes 16,064 years for the 500g of radium to decay to 5g.

How long will it take for 500g of radium to decay to 5g?

Here we have the decay equation:

[tex]Q(t) = Q_0*e^{-k*t}[/tex]

Where Q₀ is the initial amount, and k is the decay constant.

We know that:

Q₀ = 500g

k = 0.00043

And we want to find the value of t such that Q(t) = 5g, so we need to solve:

[tex]5 = 500*e^{-0.00043*t}\\\\5/500 = e^{-0.00043*t}\\\\0.001 = e^{-0.00043*t}[/tex]

Now we can apply the natural logarithm in both sides:

[tex]ln(0.001) = ln(e^{-0.00043*t})\\\\ln(0.001) = -0.00043*t\\\\\frac{ln(0.001)}{-0.00043} = t = 16,064.5[/tex]

So it takes 16,064 years for the 500g of radium to decay to 5g.

If you want to learn more about decays:

https://brainly.com/question/7920039

#SPJ1

PLEASE HELP!!! I will give brainliest pleasee help. each question has to be solved, there are no options for answers.

Answers

The intersecting secant theorem states the relationship between the two intersecting secants of the same circle. The given problems can be solved using the intersecting secant theorem.

What is Intersecting Secant Theorem?

When two line secants of a circle intersect each other outside the circle, the circle divides the secants into two segments such that the product of the outside segment and the length of the secant are equal to the product of the outside segment other secant and its length.

a(a+b)=c(c+d)

1.)

6(x+6) = 5(5+x+3)

6x + 36 = 25 + 5x + 15

x = 4

2.)

4(2x+4)=5(5+x)

8x + 16 = 25 + 5x

3x = 9

x = 3

3.)

8x(6x+8x) = 7(9+7)

8x(14x) = 112

112x² = 112

x = 1

4.)

(x+3)² = 16(x-3)

x² + 9 + 6x = 16x - 48

x² - 10x - 57 = 0

x = 14.0554

Learn more about Secant:

https://brainly.com/question/10128640

#SPJ1

Suppose the u.s. government put a 'special 20 percent luxury tax' on the retail price of expensive and fancy yachts in order to collect more taxes from boat owners. assume the price elasticity for these yachts is elastic at 2.50. conclusion: we can probably expect that yacht sales will go down and the government will not collect lots of new tax revenues.

Answers

True. We can probably expect that yacht sales will go down and the government will not collect lots of new tax revenues.

It is given that the US government has put a 20% luxury tax and the price elasticity for these yachts is elastic at 2.50.

Due to the new tax on luxury goods, it can be estimated that the sales of yachts go down as the maximum retail price of the yachts will become more expensive. This can mean that fewer people than before will be able to afford the yachts. Due to fewer sales, the government will not have enough tax revenues.

To learn more about price elasticity visit: https://brainly.com/question/4610585

#SPJ4

Line / has a slope of. The line through which of the following pair of points is
perpendicular to /?
A. (4, 6), (2, 2) B. (8,8), (2, 4)
C. (8,2), (2,4)
D. (6,2), (2, 4)

Answers

The slope of a line.

Let [tex]A(x_A,\ y_A),\ B(x_B,\ y_B)[/tex]. Then a slope of the line AB represent the formula:

[tex]m=\dfrac{y_B-y_A}{x_B-x_A}[/tex]

Substitute the coordinate od the points to the formula of a slope.

[tex]A.\\(4,\ 6),\ (2,\ 2)\\\\m=\dfrac{2-6}{2-4}=\dfrac{-4}{-2}\\\\\huge\boxed{m=2}[/tex]

[tex]B.\\(8,\ 8),\ (2,\ 4)\\\\m=\dfrac{4-8}{2-8}=\dfrac{-4}{-6}\\\\\huge\boxed{m=\dfrac{2}{3}}[/tex]

[tex]C.\\(8,\ 2),\ (2,\ 4)\\\\m=\dfrac{4-2}{2-8}=\dfrac{2}{-6}\\\\\huge\boxed{m=-\dfrac{1}{3}}[/tex]

[tex]D.\\(6,\ 2),\ (2,\ 4)\\\\m=\dfrac{4-2}{2-6}=\dfrac{2}{-4}\\\\\huge\boxed{m=-\dfrac{1}{2}}[/tex]

I need a written answer for all three questions please.

Answers

The values of the trigonometry ratios are:

cos α = - 5/13 and cot α = 5/12cot α = -5/12 and sec α = 13/5

How to solve the trigonometry ratios?

1: sin α = -12/13 and tan α > 0, find cos α and cot α

Because tan α > 0, then it means that cos α and sin α are negative

So, we have:

sin²α + cos²α = 1

Substitute sin α = -12/13

(-12/13)² + cos²α = 1

This gives

cos²α = 1 - (-12/13)²

Evaluate the squares

cos²α = 1 - 144/169

Evaluate the difference

cos²α = 25/169

Take the square root of both sides

cos α = - 5/13

The cotangent ratio is represented as:

cot α = cos α/sin α

This gives

cot α = (-5/13)/(-12/13)

Evaluate

cot α = 5/12

Hence, cos α = - 5/13 and cot α = 5/12

2: tan α = -12/5 for α in quadrant IV, find sec α and cot α

Because α is in quadrant IV, then it means that sec α is positive

cot α = 1/tan α

This gives

cot α = 1/(-12/5)

Evaluate

cot α = -5/12

Also, we have:

sec²α = 1 + tan²α

Substitute tan α = -12/5

sec²α = 1 + (-12/5)²

Evaluate the squares

sec²α = 1 + 144/25

Evaluate the sum

sec²α = 169/25

Take the square root of both sides

sec α = 13/5

Hence, cot α = -5/12 and sec α = 13/5

Read more about trigonometry ratios at:

https://brainly.com/question/11967894

#SPJ1

Other Questions
The drink you usually buy is on sale, the price has been reduced by $2 per can, this means you can get 28 cans for the same price that you usually pay for 20 cans, determine the original price if the government imposes a per-unit tax on sales of an industry's product, then we would expect What is importance to support slough PLEASE answer ASP.How is the strength of an electrostatic force determined? You own a portfolio that has $1,720 invested in Stock A and $3,470 invested in Stock B. The expected returns on these stocks are 13.7 percent and 8.0 percent, respectively. What is the expected return on the portfolio Which statement is accurate about Japan and Korea due to their close proximity 9.25(18.4-2.1.2)Please hurry I need to Finnish 2 more pages today Challenge Anyone?...... Which of the following is equivalent to the expression, i^39?O A. -1OB. 1O C. -iO D. i _____ is a force which is applied to an object by another object or a person. __________ is a force that always opposes the motion of an object and is frequently ignored. a. Tensional force, friction c. Applied force, air resistance b. Friction, applied force d. None of the above. Please select the best answer from the choices provided A B C D We calculate the value of the hypotenuse of the right triangle whose legs measure 3 and 4cm respectivelyHelp please it's due today What did johannes kepler contribute to the study of planets? he formulated the universal law of gravitation. he identified the first moons of jupiter. he collected detailed data that led to the proposal of the heliocentric model. he provided mathematical support for the heliocentric model. How was Rockefeller able to build his monopoly across the oil industry?He bought up oil refineries, cut costs, and reinvested his profits in other refineries.He confined his business to Ohio so he could buy all the refineries there.He found newer and cheaper ways to refine oil, increasing his profits.He began to sell kerosene as well as oil, expanding his market. Solve (2465)8*(465)8 given the issues of the historical period, why were logical fallacies used to persuade american people during world war ii? a americans were told the complete truth about the war during the world war ii era. b. logical fallacies were only used by the united states' enemies during world war ii as a way to create more fear in their citizens about the dangers of the united states military. c. logical fallacies were truthful statements used by american soldiers while fighting in war zones to help persuade the enemy soldiers to surrender to the united states. d logical fallacies played on the fears and emotions of the american people, which allowed for manipulating them into service. americans were desperately needed during wartime to continue production that sustained the overseas military as well as the nation's demands for labor, goods, and food. 25.A ladder stands against a wall at an angle of 45 to the floor. The foot of the ladder is 3 mfrom the wall. If the ladder slides 1 m down the wall, then the angle between the ladderand the floor is such that: why is a salt added to a password that is being stored in a database? The point (0, 0) is a solution to which of these inequalities?O Ay+7 The formula for calculating the amount of interest charged on a loan is:interest = [principal x rate] of interest x timeWhich Python statement correctly performs the interest calculation? What is a good personal objective to apply a SWOT Analysis to?